No one who works at Leila's Electronics has received both a poor performance evaluation and a raise. Lester has not r...

Maroun on March 28, 2019

Please help

Can you run through this for me? I am very confused with the correct answer choice, and I am not sure how my diagrams are incorrect

Reply
Create a free account to read and take part in forum discussions.

Already have an account? log in

shunhe on December 28, 2019

Hi @Maroun,

Here's one way to diagram it, which is generally a good approach to take a match the flaw question. I'll diagram the stimulus first. An implicit premise here is that Lester works at Leila's electronics; it would've been better if this had been made explicit, but by the time we get to the answer choices it's pretty obvious.

WLE = Work at Leila's electronics
RPPE = Receive poor performance evaluation
RR = receive raise

P1) WLE - > ~(RPPE & RR)
We can rewrite this as
WLE - > ~RPPE v ~RR

P2) ~RR

Conclusion: RPPE.

To translate what's going on into English, since we know that Lester works at Leila's electronic, he can't have gotten both a poor performance evaluation and a raise. And we know he didn't get a raise, so we conclude that he received a poor performance evaluation. But the problem with this logic is the following: it is completely possible that Lester could not have gotten a raise and not have gotten a poor performance evaluation. Remember that the logical "OR" is inclusive and so both conditions can be true. Even if WLE - > ~RR & ~RPPE, (P1) is still true. The answer choice we're looking for has to match this solution, and after going through them, we can see that (D) commits the same logical fallacy. Diagramming (D), we get

P1) Lives in house - > ~(Own & Pay rent)
Which we rewrite as
Lives in house - > ~Own v ~Pay rent

P2) ~Own house

Conclusion: Pay rent

And we can immediately see that this parallels the structure of the stimulus. Hope this helps! If anything was unclear, feel free to ask further questions.